Prove $x equiv a pmod{p}$ and $x equiv a pmod{q}$ then $x equiv apmod{pq}$












3












$begingroup$


$p$ and $q$ are distinct primes.



Where can I start with this proof?
It looks similar to the Chinese Remainder Theorem, but that deals with two different a values.










share|cite|improve this question











$endgroup$








  • 1




    $begingroup$
    but that deals with two different $a$ values - What makes you think that?
    $endgroup$
    – anon
    Sep 3 '12 at 15:59










  • $begingroup$
    Because for the explanations of Chinese Remainder Theorem I've read, they use something like a = x (mod p) and a = y (mod p)
    $endgroup$
    – Takkun
    Sep 3 '12 at 16:03






  • 3




    $begingroup$
    A couple of variables denoted with two different letters may take on two different values, but not necessarily unless they are explicitly stated to have distinct values.
    $endgroup$
    – anon
    Sep 3 '12 at 16:04


















3












$begingroup$


$p$ and $q$ are distinct primes.



Where can I start with this proof?
It looks similar to the Chinese Remainder Theorem, but that deals with two different a values.










share|cite|improve this question











$endgroup$








  • 1




    $begingroup$
    but that deals with two different $a$ values - What makes you think that?
    $endgroup$
    – anon
    Sep 3 '12 at 15:59










  • $begingroup$
    Because for the explanations of Chinese Remainder Theorem I've read, they use something like a = x (mod p) and a = y (mod p)
    $endgroup$
    – Takkun
    Sep 3 '12 at 16:03






  • 3




    $begingroup$
    A couple of variables denoted with two different letters may take on two different values, but not necessarily unless they are explicitly stated to have distinct values.
    $endgroup$
    – anon
    Sep 3 '12 at 16:04
















3












3








3





$begingroup$


$p$ and $q$ are distinct primes.



Where can I start with this proof?
It looks similar to the Chinese Remainder Theorem, but that deals with two different a values.










share|cite|improve this question











$endgroup$




$p$ and $q$ are distinct primes.



Where can I start with this proof?
It looks similar to the Chinese Remainder Theorem, but that deals with two different a values.







number-theory






share|cite|improve this question















share|cite|improve this question













share|cite|improve this question




share|cite|improve this question








edited Sep 3 '12 at 16:08









Thomas Russell

7,89632552




7,89632552










asked Sep 3 '12 at 15:57









TakkunTakkun

268311




268311








  • 1




    $begingroup$
    but that deals with two different $a$ values - What makes you think that?
    $endgroup$
    – anon
    Sep 3 '12 at 15:59










  • $begingroup$
    Because for the explanations of Chinese Remainder Theorem I've read, they use something like a = x (mod p) and a = y (mod p)
    $endgroup$
    – Takkun
    Sep 3 '12 at 16:03






  • 3




    $begingroup$
    A couple of variables denoted with two different letters may take on two different values, but not necessarily unless they are explicitly stated to have distinct values.
    $endgroup$
    – anon
    Sep 3 '12 at 16:04
















  • 1




    $begingroup$
    but that deals with two different $a$ values - What makes you think that?
    $endgroup$
    – anon
    Sep 3 '12 at 15:59










  • $begingroup$
    Because for the explanations of Chinese Remainder Theorem I've read, they use something like a = x (mod p) and a = y (mod p)
    $endgroup$
    – Takkun
    Sep 3 '12 at 16:03






  • 3




    $begingroup$
    A couple of variables denoted with two different letters may take on two different values, but not necessarily unless they are explicitly stated to have distinct values.
    $endgroup$
    – anon
    Sep 3 '12 at 16:04










1




1




$begingroup$
but that deals with two different $a$ values - What makes you think that?
$endgroup$
– anon
Sep 3 '12 at 15:59




$begingroup$
but that deals with two different $a$ values - What makes you think that?
$endgroup$
– anon
Sep 3 '12 at 15:59












$begingroup$
Because for the explanations of Chinese Remainder Theorem I've read, they use something like a = x (mod p) and a = y (mod p)
$endgroup$
– Takkun
Sep 3 '12 at 16:03




$begingroup$
Because for the explanations of Chinese Remainder Theorem I've read, they use something like a = x (mod p) and a = y (mod p)
$endgroup$
– Takkun
Sep 3 '12 at 16:03




3




3




$begingroup$
A couple of variables denoted with two different letters may take on two different values, but not necessarily unless they are explicitly stated to have distinct values.
$endgroup$
– anon
Sep 3 '12 at 16:04






$begingroup$
A couple of variables denoted with two different letters may take on two different values, but not necessarily unless they are explicitly stated to have distinct values.
$endgroup$
– anon
Sep 3 '12 at 16:04












3 Answers
3






active

oldest

votes


















6












$begingroup$

Hint $ $ Below are few proofs of this constant-case CCRT of Chinese Remainder Theorem (CRT). The first three proofs work for for arbitrary coprime naturals $rm,p,q.$



$rm(1) x equiv apmod {pq}:$ is clearly a solution, and the solution is $color{#C00}{unique}$ mod $rm,pq,$ by CRT.



$rm(2) p,q:|:x!-!aiff lcm(p,q):|:x!-!a:$ by the Universal Property of $rm lcm$.
$qquad! $ Further $rm:(p,q)=1iff:lcm(p,q) = pq,,$ by this answer.



$(3) , $ By Euclid's Lemma: $rm:(p,q)=1, p:|:qn =:x!-!a:Rightarrow:p:|:n:Rightarrow:pq:|:nq = x!-!a.$



$rm(4) , $ Since the prime factorization of $rm,x!-!a,$ is $color{#C00}{unique}$, and the prime $rm,p,$ occurs in one factorization, and the distinct prime $rm,q,$ occurs in another, both factorizations are the same up to order, hence contain both $rm,p,$ and $rm,q,:$ hence have $rm,pq,$ as a divisor.



Remark $ $ This constant-case optimization of CRT arises frequently in practice so is well-worth memorizing, e.g. see some prior posts for many examples.



Quite frequently, $color{#C00}{uniqueness} theorems$ provide powerful tools for proving equalities.






share|cite|improve this answer











$endgroup$













  • $begingroup$
    in (2) do you mean to say $gcd(p, q) = 1 iff lcm(p, q) = pq$
    $endgroup$
    – CodeKingPlusPlus
    Sep 23 '12 at 23:17










  • $begingroup$
    @Code Yes, $rm:(x,y):$ means $rm:gcd(x,y):$ in number theory (common notation).
    $endgroup$
    – Bill Dubuque
    Sep 23 '12 at 23:37



















4












$begingroup$

Let $[A,B]=lcm(A,B)$ and $(A,B)=gcd(A,B)$



If $p,q$ are different integers, $pmid(x-a)$ and $qmid(x-a)implies [p,q]mid(x-a)$



We know $[p,q]cdot (p,q)=pcdot q$



If $(p,q)=1, [p,q]=pcdot q$



If $p,q$ are distinct primes, $(p,q)=1$






share|cite|improve this answer











$endgroup$





















    3












    $begingroup$

    Let $y=x-a$. We want to show that if $p$ divides $y$ and $q$ divides $y$ then $pq$ divides $y$.



    Since $p$ divides $y$, we have $y=pz$ for some $z$. Thus $q$ divides $pz$. Since $q$ is prime, this implies $q$ divides $p$ or $q$ divides $z$. But $q$ cannot divide $p$, so $q$ divides $z$. Suppose that $z=qw$. Then $y=pqw$.






    share|cite|improve this answer











    $endgroup$













      Your Answer





      StackExchange.ifUsing("editor", function () {
      return StackExchange.using("mathjaxEditing", function () {
      StackExchange.MarkdownEditor.creationCallbacks.add(function (editor, postfix) {
      StackExchange.mathjaxEditing.prepareWmdForMathJax(editor, postfix, [["$", "$"], ["\\(","\\)"]]);
      });
      });
      }, "mathjax-editing");

      StackExchange.ready(function() {
      var channelOptions = {
      tags: "".split(" "),
      id: "69"
      };
      initTagRenderer("".split(" "), "".split(" "), channelOptions);

      StackExchange.using("externalEditor", function() {
      // Have to fire editor after snippets, if snippets enabled
      if (StackExchange.settings.snippets.snippetsEnabled) {
      StackExchange.using("snippets", function() {
      createEditor();
      });
      }
      else {
      createEditor();
      }
      });

      function createEditor() {
      StackExchange.prepareEditor({
      heartbeatType: 'answer',
      autoActivateHeartbeat: false,
      convertImagesToLinks: true,
      noModals: true,
      showLowRepImageUploadWarning: true,
      reputationToPostImages: 10,
      bindNavPrevention: true,
      postfix: "",
      imageUploader: {
      brandingHtml: "Powered by u003ca class="icon-imgur-white" href="https://imgur.com/"u003eu003c/au003e",
      contentPolicyHtml: "User contributions licensed under u003ca href="https://creativecommons.org/licenses/by-sa/3.0/"u003ecc by-sa 3.0 with attribution requiredu003c/au003e u003ca href="https://stackoverflow.com/legal/content-policy"u003e(content policy)u003c/au003e",
      allowUrls: true
      },
      noCode: true, onDemand: true,
      discardSelector: ".discard-answer"
      ,immediatelyShowMarkdownHelp:true
      });


      }
      });














      draft saved

      draft discarded


















      StackExchange.ready(
      function () {
      StackExchange.openid.initPostLogin('.new-post-login', 'https%3a%2f%2fmath.stackexchange.com%2fquestions%2f190514%2fprove-x-equiv-a-pmodp-and-x-equiv-a-pmodq-then-x-equiv-a-pmodpq%23new-answer', 'question_page');
      }
      );

      Post as a guest















      Required, but never shown

























      3 Answers
      3






      active

      oldest

      votes








      3 Answers
      3






      active

      oldest

      votes









      active

      oldest

      votes






      active

      oldest

      votes









      6












      $begingroup$

      Hint $ $ Below are few proofs of this constant-case CCRT of Chinese Remainder Theorem (CRT). The first three proofs work for for arbitrary coprime naturals $rm,p,q.$



      $rm(1) x equiv apmod {pq}:$ is clearly a solution, and the solution is $color{#C00}{unique}$ mod $rm,pq,$ by CRT.



      $rm(2) p,q:|:x!-!aiff lcm(p,q):|:x!-!a:$ by the Universal Property of $rm lcm$.
      $qquad! $ Further $rm:(p,q)=1iff:lcm(p,q) = pq,,$ by this answer.



      $(3) , $ By Euclid's Lemma: $rm:(p,q)=1, p:|:qn =:x!-!a:Rightarrow:p:|:n:Rightarrow:pq:|:nq = x!-!a.$



      $rm(4) , $ Since the prime factorization of $rm,x!-!a,$ is $color{#C00}{unique}$, and the prime $rm,p,$ occurs in one factorization, and the distinct prime $rm,q,$ occurs in another, both factorizations are the same up to order, hence contain both $rm,p,$ and $rm,q,:$ hence have $rm,pq,$ as a divisor.



      Remark $ $ This constant-case optimization of CRT arises frequently in practice so is well-worth memorizing, e.g. see some prior posts for many examples.



      Quite frequently, $color{#C00}{uniqueness} theorems$ provide powerful tools for proving equalities.






      share|cite|improve this answer











      $endgroup$













      • $begingroup$
        in (2) do you mean to say $gcd(p, q) = 1 iff lcm(p, q) = pq$
        $endgroup$
        – CodeKingPlusPlus
        Sep 23 '12 at 23:17










      • $begingroup$
        @Code Yes, $rm:(x,y):$ means $rm:gcd(x,y):$ in number theory (common notation).
        $endgroup$
        – Bill Dubuque
        Sep 23 '12 at 23:37
















      6












      $begingroup$

      Hint $ $ Below are few proofs of this constant-case CCRT of Chinese Remainder Theorem (CRT). The first three proofs work for for arbitrary coprime naturals $rm,p,q.$



      $rm(1) x equiv apmod {pq}:$ is clearly a solution, and the solution is $color{#C00}{unique}$ mod $rm,pq,$ by CRT.



      $rm(2) p,q:|:x!-!aiff lcm(p,q):|:x!-!a:$ by the Universal Property of $rm lcm$.
      $qquad! $ Further $rm:(p,q)=1iff:lcm(p,q) = pq,,$ by this answer.



      $(3) , $ By Euclid's Lemma: $rm:(p,q)=1, p:|:qn =:x!-!a:Rightarrow:p:|:n:Rightarrow:pq:|:nq = x!-!a.$



      $rm(4) , $ Since the prime factorization of $rm,x!-!a,$ is $color{#C00}{unique}$, and the prime $rm,p,$ occurs in one factorization, and the distinct prime $rm,q,$ occurs in another, both factorizations are the same up to order, hence contain both $rm,p,$ and $rm,q,:$ hence have $rm,pq,$ as a divisor.



      Remark $ $ This constant-case optimization of CRT arises frequently in practice so is well-worth memorizing, e.g. see some prior posts for many examples.



      Quite frequently, $color{#C00}{uniqueness} theorems$ provide powerful tools for proving equalities.






      share|cite|improve this answer











      $endgroup$













      • $begingroup$
        in (2) do you mean to say $gcd(p, q) = 1 iff lcm(p, q) = pq$
        $endgroup$
        – CodeKingPlusPlus
        Sep 23 '12 at 23:17










      • $begingroup$
        @Code Yes, $rm:(x,y):$ means $rm:gcd(x,y):$ in number theory (common notation).
        $endgroup$
        – Bill Dubuque
        Sep 23 '12 at 23:37














      6












      6








      6





      $begingroup$

      Hint $ $ Below are few proofs of this constant-case CCRT of Chinese Remainder Theorem (CRT). The first three proofs work for for arbitrary coprime naturals $rm,p,q.$



      $rm(1) x equiv apmod {pq}:$ is clearly a solution, and the solution is $color{#C00}{unique}$ mod $rm,pq,$ by CRT.



      $rm(2) p,q:|:x!-!aiff lcm(p,q):|:x!-!a:$ by the Universal Property of $rm lcm$.
      $qquad! $ Further $rm:(p,q)=1iff:lcm(p,q) = pq,,$ by this answer.



      $(3) , $ By Euclid's Lemma: $rm:(p,q)=1, p:|:qn =:x!-!a:Rightarrow:p:|:n:Rightarrow:pq:|:nq = x!-!a.$



      $rm(4) , $ Since the prime factorization of $rm,x!-!a,$ is $color{#C00}{unique}$, and the prime $rm,p,$ occurs in one factorization, and the distinct prime $rm,q,$ occurs in another, both factorizations are the same up to order, hence contain both $rm,p,$ and $rm,q,:$ hence have $rm,pq,$ as a divisor.



      Remark $ $ This constant-case optimization of CRT arises frequently in practice so is well-worth memorizing, e.g. see some prior posts for many examples.



      Quite frequently, $color{#C00}{uniqueness} theorems$ provide powerful tools for proving equalities.






      share|cite|improve this answer











      $endgroup$



      Hint $ $ Below are few proofs of this constant-case CCRT of Chinese Remainder Theorem (CRT). The first three proofs work for for arbitrary coprime naturals $rm,p,q.$



      $rm(1) x equiv apmod {pq}:$ is clearly a solution, and the solution is $color{#C00}{unique}$ mod $rm,pq,$ by CRT.



      $rm(2) p,q:|:x!-!aiff lcm(p,q):|:x!-!a:$ by the Universal Property of $rm lcm$.
      $qquad! $ Further $rm:(p,q)=1iff:lcm(p,q) = pq,,$ by this answer.



      $(3) , $ By Euclid's Lemma: $rm:(p,q)=1, p:|:qn =:x!-!a:Rightarrow:p:|:n:Rightarrow:pq:|:nq = x!-!a.$



      $rm(4) , $ Since the prime factorization of $rm,x!-!a,$ is $color{#C00}{unique}$, and the prime $rm,p,$ occurs in one factorization, and the distinct prime $rm,q,$ occurs in another, both factorizations are the same up to order, hence contain both $rm,p,$ and $rm,q,:$ hence have $rm,pq,$ as a divisor.



      Remark $ $ This constant-case optimization of CRT arises frequently in practice so is well-worth memorizing, e.g. see some prior posts for many examples.



      Quite frequently, $color{#C00}{uniqueness} theorems$ provide powerful tools for proving equalities.







      share|cite|improve this answer














      share|cite|improve this answer



      share|cite|improve this answer








      edited Feb 7 at 16:34

























      answered Sep 3 '12 at 16:18









      Bill DubuqueBill Dubuque

      213k29195654




      213k29195654












      • $begingroup$
        in (2) do you mean to say $gcd(p, q) = 1 iff lcm(p, q) = pq$
        $endgroup$
        – CodeKingPlusPlus
        Sep 23 '12 at 23:17










      • $begingroup$
        @Code Yes, $rm:(x,y):$ means $rm:gcd(x,y):$ in number theory (common notation).
        $endgroup$
        – Bill Dubuque
        Sep 23 '12 at 23:37


















      • $begingroup$
        in (2) do you mean to say $gcd(p, q) = 1 iff lcm(p, q) = pq$
        $endgroup$
        – CodeKingPlusPlus
        Sep 23 '12 at 23:17










      • $begingroup$
        @Code Yes, $rm:(x,y):$ means $rm:gcd(x,y):$ in number theory (common notation).
        $endgroup$
        – Bill Dubuque
        Sep 23 '12 at 23:37
















      $begingroup$
      in (2) do you mean to say $gcd(p, q) = 1 iff lcm(p, q) = pq$
      $endgroup$
      – CodeKingPlusPlus
      Sep 23 '12 at 23:17




      $begingroup$
      in (2) do you mean to say $gcd(p, q) = 1 iff lcm(p, q) = pq$
      $endgroup$
      – CodeKingPlusPlus
      Sep 23 '12 at 23:17












      $begingroup$
      @Code Yes, $rm:(x,y):$ means $rm:gcd(x,y):$ in number theory (common notation).
      $endgroup$
      – Bill Dubuque
      Sep 23 '12 at 23:37




      $begingroup$
      @Code Yes, $rm:(x,y):$ means $rm:gcd(x,y):$ in number theory (common notation).
      $endgroup$
      – Bill Dubuque
      Sep 23 '12 at 23:37











      4












      $begingroup$

      Let $[A,B]=lcm(A,B)$ and $(A,B)=gcd(A,B)$



      If $p,q$ are different integers, $pmid(x-a)$ and $qmid(x-a)implies [p,q]mid(x-a)$



      We know $[p,q]cdot (p,q)=pcdot q$



      If $(p,q)=1, [p,q]=pcdot q$



      If $p,q$ are distinct primes, $(p,q)=1$






      share|cite|improve this answer











      $endgroup$


















        4












        $begingroup$

        Let $[A,B]=lcm(A,B)$ and $(A,B)=gcd(A,B)$



        If $p,q$ are different integers, $pmid(x-a)$ and $qmid(x-a)implies [p,q]mid(x-a)$



        We know $[p,q]cdot (p,q)=pcdot q$



        If $(p,q)=1, [p,q]=pcdot q$



        If $p,q$ are distinct primes, $(p,q)=1$






        share|cite|improve this answer











        $endgroup$
















          4












          4








          4





          $begingroup$

          Let $[A,B]=lcm(A,B)$ and $(A,B)=gcd(A,B)$



          If $p,q$ are different integers, $pmid(x-a)$ and $qmid(x-a)implies [p,q]mid(x-a)$



          We know $[p,q]cdot (p,q)=pcdot q$



          If $(p,q)=1, [p,q]=pcdot q$



          If $p,q$ are distinct primes, $(p,q)=1$






          share|cite|improve this answer











          $endgroup$



          Let $[A,B]=lcm(A,B)$ and $(A,B)=gcd(A,B)$



          If $p,q$ are different integers, $pmid(x-a)$ and $qmid(x-a)implies [p,q]mid(x-a)$



          We know $[p,q]cdot (p,q)=pcdot q$



          If $(p,q)=1, [p,q]=pcdot q$



          If $p,q$ are distinct primes, $(p,q)=1$







          share|cite|improve this answer














          share|cite|improve this answer



          share|cite|improve this answer








          edited Sep 3 '12 at 16:06

























          answered Sep 3 '12 at 16:01









          lab bhattacharjeelab bhattacharjee

          227k15158278




          227k15158278























              3












              $begingroup$

              Let $y=x-a$. We want to show that if $p$ divides $y$ and $q$ divides $y$ then $pq$ divides $y$.



              Since $p$ divides $y$, we have $y=pz$ for some $z$. Thus $q$ divides $pz$. Since $q$ is prime, this implies $q$ divides $p$ or $q$ divides $z$. But $q$ cannot divide $p$, so $q$ divides $z$. Suppose that $z=qw$. Then $y=pqw$.






              share|cite|improve this answer











              $endgroup$


















                3












                $begingroup$

                Let $y=x-a$. We want to show that if $p$ divides $y$ and $q$ divides $y$ then $pq$ divides $y$.



                Since $p$ divides $y$, we have $y=pz$ for some $z$. Thus $q$ divides $pz$. Since $q$ is prime, this implies $q$ divides $p$ or $q$ divides $z$. But $q$ cannot divide $p$, so $q$ divides $z$. Suppose that $z=qw$. Then $y=pqw$.






                share|cite|improve this answer











                $endgroup$
















                  3












                  3








                  3





                  $begingroup$

                  Let $y=x-a$. We want to show that if $p$ divides $y$ and $q$ divides $y$ then $pq$ divides $y$.



                  Since $p$ divides $y$, we have $y=pz$ for some $z$. Thus $q$ divides $pz$. Since $q$ is prime, this implies $q$ divides $p$ or $q$ divides $z$. But $q$ cannot divide $p$, so $q$ divides $z$. Suppose that $z=qw$. Then $y=pqw$.






                  share|cite|improve this answer











                  $endgroup$



                  Let $y=x-a$. We want to show that if $p$ divides $y$ and $q$ divides $y$ then $pq$ divides $y$.



                  Since $p$ divides $y$, we have $y=pz$ for some $z$. Thus $q$ divides $pz$. Since $q$ is prime, this implies $q$ divides $p$ or $q$ divides $z$. But $q$ cannot divide $p$, so $q$ divides $z$. Suppose that $z=qw$. Then $y=pqw$.







                  share|cite|improve this answer














                  share|cite|improve this answer



                  share|cite|improve this answer








                  edited Sep 3 '12 at 16:30

























                  answered Sep 3 '12 at 16:08









                  André NicolasAndré Nicolas

                  454k36432819




                  454k36432819






























                      draft saved

                      draft discarded




















































                      Thanks for contributing an answer to Mathematics Stack Exchange!


                      • Please be sure to answer the question. Provide details and share your research!

                      But avoid



                      • Asking for help, clarification, or responding to other answers.

                      • Making statements based on opinion; back them up with references or personal experience.


                      Use MathJax to format equations. MathJax reference.


                      To learn more, see our tips on writing great answers.




                      draft saved


                      draft discarded














                      StackExchange.ready(
                      function () {
                      StackExchange.openid.initPostLogin('.new-post-login', 'https%3a%2f%2fmath.stackexchange.com%2fquestions%2f190514%2fprove-x-equiv-a-pmodp-and-x-equiv-a-pmodq-then-x-equiv-a-pmodpq%23new-answer', 'question_page');
                      }
                      );

                      Post as a guest















                      Required, but never shown





















































                      Required, but never shown














                      Required, but never shown












                      Required, but never shown







                      Required, but never shown

































                      Required, but never shown














                      Required, but never shown












                      Required, but never shown







                      Required, but never shown







                      Popular posts from this blog

                      Can a sorcerer learn a 5th-level spell early by creating spell slots using the Font of Magic feature?

                      Does disintegrating a polymorphed enemy still kill it after the 2018 errata?

                      A Topological Invariant for $pi_3(U(n))$